Where did the formula for summing finite geometric series come from? Suppose the geometric series has first term a₁ and constant ratio r , so that S n= a₁ + a₁r+ a₁ r²+ . . +a₁ rⁿ⁻¹

a. Show that r S n = a₁r+ a₁ r²+ a₁ r³+ . . . . +a₁rⁿ .

Answers

Answer 1

The total of a geometric series is an unlimited number of terms with a fixed ratio between them.

If S n= a₁ + a₁r+ a₁ r²+ . . +a₁ rⁿ⁻¹ then rSn = a₁r + a₁ r²+ . . +a₁ rⁿ⁻¹+a₁ rⁿ.

What is the formula for geometric series?

The sum of a geometric series Sn  is given by:

[tex]$S_n=a(\frac{1-r^{n} }{1-r} )[/tex]

Given:

Sum of first n terms of the Geometric progression be

Sn = a₁ + a₁r + a₁ r² + . . + a₁ rⁿ⁻¹  ---(1)

On multiplying by r on both sides,

r Sn = a₁r + a₁ r² + . . + a₁ rⁿ⁻¹ + a₁ rⁿ  ----(2)

So, (2) − (1) gives,

[tex]rS_n-S_n=ar^{n} -a[/tex]

[tex]S_n(r-1)=a(r^{n} -1)[/tex]

Therefore, the sum of n terms of a GP is:

[tex]$S_n=a(\frac{1-r^{n} }{1-r} )[/tex]

Let the expression be r S n = a₁r + a₁ r² + a₁ r³ + . . . . + a₁rⁿ

Sum of first n terms of the Geometric progression is:

Sn = a₁ + a₁r + a₁ r² + . . + a₁ rⁿ⁻¹  

On multiplying by r on both sides, we get

rSn = a₁r + a₁ r²+ . . +a₁ rⁿ⁻¹+a₁ rⁿ  

Therefore, rSn = a₁r + a₁ r²+ . . +a₁ rⁿ⁻¹+a₁ rⁿ  

To learn more about geometric series, refer:

https://brainly.com/question/24643676

#SPJ4


Related Questions

determine whether the following graph represent a function.

1. one to one function
2. function but not one to one
3. not a function

Answers

1. One to one function


Using a calculator, what is the solution of 1080=15³ˣ⁻⁴ ? Round the answer to the nearest hundredth.

Answers

The solution of the given equation 1080 = 15³ˣ⁻⁴ is (x = 2.19), round to the nearest hundredth.

What is defined as the logarithmic functions?

A logarithmic term represents a number in logarithmic form. A log term is another name for it.

A quantity can also be the sum of two or even more quantities. As a result, a term can consist of the product of two or more quantities, one of which must be in logarithmic form. For such cases, this same terms are known as log terms.

A quantity can also be defined as the quotient of two quantities. But, if a term includes a quotient of quantities, or at least one of the quantities can also be expressed in log form, this same terms are known as log terms.

Quotient Rule: ㏒(A/B) = ㏒A - ㏒BProduct Rule : ㏒A + ㏒B = ㏒(AB)Power Rule: n㏒A = ㏒AⁿBase Rule = ㏒ₐb = ㏒ₓb/㏒ₓaExponent Rule = e∧lnx = x

Now, consider the given equation given in the question;

1080=15³ˣ⁻⁴

Take log on the both sides;

㏒1080 = ㏒15³ˣ⁻⁴

Using the power rule on the second side of the equation;

㏒1080 = (3x - 4)㏒15

Taking log on one side and other variables on other side.

3x - 4 = ㏒1080/㏒15

Use calculator to find the values of both log.

3x - 4 = 2.57

3x = 6.57

x = 2.19

Therefore, the solution of the given function 1080=15³ˣ⁻⁴ is (x = 2.19), rounded up to nearest hundredth.

To know more about the logarithmic functions, here

https://brainly.com/question/1695836

#SPJ4

You can find the equation of a line through two points even if one point is not the y -intercept. Find the slope m of the line passing through the two points.Using either point, substitute for x, y , and m into y=m x+b . Solve for b and rewrite y=m x+b for the values of m and b . Write an equation in slope-intercept form for the line passing through each pair of points.

b. (-4,16) and (3,-5)

Answers

16 = -4(-3) + 4 is an equation in slope-intercept form for the line passing through points (-4,16) and (3,-5)

What is a slope of line?

A line's steepness and direction are determined by the slope of the line. Without actually using a compass, determining a line's slope in a coordinate plane allows one to anticipate whether a line is parallel, perpendicular, or not.

The change in a line's y coordinate relative to its change in x coordinate is referred to as the line's slope. Δy is the net change in the y coordinate, while Δx is the net change in the x coordinate.  

Therefore,

m =  y₂-y₁/x₂-x₁ (where m denotes the slope) can be used to express how the y coordinate changes in relation to the x coordinate.

Be aware that tan ∅ = y/x

We also refer to this tan as the line's slope.

We have been asked to write an equation in slope-intercept form for the line passing through (-4,16) and (3,-5)

First we will find the slope(m)

We know that slope = m = [tex]\frac{y_2 - y_1}{x_2 - x_1}[/tex]

For the pairs of points (-4,16) and (3,-5)

⇒ m = [tex]\frac{-5- 16}{3 - (-4)}[/tex]

⇒ m = [tex]\frac{-21}{7}[/tex]

⇒ m = -3

We know that the equation for slope(m) and y - intercept(b) of a line is given by :

y = mx + b

Here lets take equation 16 = -4(-3) + b .Here x and y is substituted with the second two points.

⇒  16 = -4(-3) + b

⇒  16 = 12 + b

⇒  b = 16 - 12

⇒  b = 4

The equation in the slope intercept is as follows:-

16 = -4(-3) + 4

Learn more about slope

brainly.com/question/16949303

#SPJ4

Jeff, Gene, and Mike need to eat an entire cake before it goes bad. The cake is cut into 8 pieces.

- Jeff could eat 6 pieces of cake in 2 hours
- Gene can eat 2 pieces of cake in 30 minutes
- Mike can eat 4 pieces of cake in 48 minutes

The pieces can be divided into smaller pieces as necessary - the rates are based on the size of pieces that result when dividing the cake into 8 equal pieces.

Answer these questions below:

1) How many pieces of cake can jeff eat in 1 hour? _________________

2) How many pieces of cake can Gene eat in 1 hour? _____________________

3) How many pieces of cake can Mike eat in 60 minutes? ____________________

4) Will it take Mike, Gene, and Jeff more than 1 hour to each eat the cake, or less that an hour to eat the cake? Explain how you know.

Answers

The pieces of cake that Jeff will eat in 1 hour is 3

The pieces of cake that Gene will eat in 1 hour is 4

The pieces of cake that Mike will eat in 1 hour is 5

It will take Mike, Gene, and Jeff more than 1 hour to each eat the cake

How many pieces of cake would they eat in 1 hour?

In order to determine the pieces of cake that Jeff can eat in 1 hour, divide 6 by 2.

6/2 = 3 pieces of cake

In order to determine the pieces of cake that Gene can eat in 1 hour, multiply 2 pieces of cake by 2.

2 x 2 = 4 pieces of cake

In order to determine the pieces of cake that Gene can eat in 1 hour, multiply 4 pieces of cake by 60 minutes and divide the result by 48 minutes.

(4 x 60) / 48 = 5 pieces of cake

It will take Mike, Gene, and Jeff more than 1 hour to each eat the cake, because they all eat less than 8 slices of cake in one hour.

To learn more about division, please check: https://brainly.com/question/13281206

#SPJ1


Find the measures of the sides of Δ X Y Z and classify
triangle by its sides.
X(-5,9), Y(2,1), Z(-8,3)

Answers

The measures of the sides of ΔXYZ are XY = 10.6, YZ = 11.2, XZ =  6.7 and the triangle XYZ must be an acute triangle

In this questions we have been given the the coordinates of the triangle XYZ.

X(-5, 9), Y(2, 1), Z(-8, 3)

We need to find  the measures of the sides of ΔXYZ .

We find the the measures of the sides using distance formula.

XY = √[(1 - 9)² + (2 + 5)²]

XY = √[-8² + 7²]

XY = √(64 + 49)

XY = √(113)

XY = 10.6

Now side YZ

YZ = √[(3 - 1)² + (-8 - 2)²]

YZ = √[(2)² + (-10)²]

YZ = √4 + 100

YZ = √104

YZ = 11.2

And the length of side XZ would be,

XZ = √[(3 - 9)² + (-8 + 5)²]

XZ = √(-6² + -3²)

XZ = √(36 + 9)

XZ = √45

XZ = 6.7

We find the sum of the squares of the two smaller sides, and compare the sum to the square of the largest side.

the sum of the squares of the two smaller sides is,

= 6.7² + 10.6²

= 44.89 + 115.54

= 160.43

= 12.67²

And 11.2² = 125.44

Since the sum of the squares of the two smaller sides is greater than the square of the largest side, the triangle must be an acute triangle.

Therefore, the measures of the sides of ΔXYZ are XY = 10.6, YZ = 11.2, XZ =  6.7 and the triangle XYZ must be an acute triangle

Learn more about the triangle here:

brainly.com/question/2456591

#SPJ4

HELP ASAP!!! WILL MARK BRAINLIEST!!!

What are the coordinates of G after it is rotated 90 degrees clockwise?

Answers

the answer would be (-3,-4).

explanation-
u switch the (x, y) so it would be (y, x) but the y has to be negative since it’s clockwise and the x stays the same.

Answer:

See below

Step-by-step explanation:

Rotating CLOCK wise would result in -4,3 becoming  3,4 in the first quadrant

In order to design an appropriate touchscreen for a computer, the manufacturer has to consider the average size of the fingers and hands that will be touching the screen. One lab found that the average width of the index ( or pointing ) finger for adults is from 1.6 cm to 2 cm wide.

1. All the measurements on this page are inclusive. Explain what that means for the inequality symbols you will use.

2. Write the average width as an inequality using centimeters.

3. One centimeter = 0.39 inches. Write the average width as an inequality using inches.

Answers

The range of the average width of the index finger which is between 1.6 centimeters and 2 centimeters gives;

1. The symbols to use for the inequality are ≤ and ≥

2. The average width expressed as an inequality is; 1.6 ≤ w ≤ 2

3. The inequality for the average width in inches is 0.624 ≤ w ≤ 0.78

How can the inclusive range be expressed as an inequality?

1. With regards to working with number ranges or intervals, the term inclusive means that the endpoints of the (closed) interval are included in the range.

The inequality symbol to be used in an inclusive interval are ≤ and ≥ (the symbols < and > are used for an exclusive measurements)

2. The minimum width = 1.6 centimeters

Maximum width = 2 centimeters

The average width in centimeters, w, written as an inequality is therefore;

1.6 ≤ w ≤ 2

3. The given conversion factor is 1 cm. = 0.39 inches.

Therefore;

1.6 cm = 1.6 × 0.39 in. = 0.624 in.

2 cm = 2 × 0.39 in. = 0.78 in.

The inequality for the average width in inches is therefore;

0.624 ≤ w ≤ 0.78

Learn more about inequalities here:

https://brainly.com/question/22976364

#SPJ1

Masha bought red, green, and yellow balloons for a party. She bought 40 red balloons. The number of green balloons Masha bought is 3/5 of the number of red balloons. The number of green balloons is also 2/3 of the number of yellow balloons.

Answers

The number of green and yellow balls is 24 and 36 respectively.

Given that,
Masha bought red, green, and yellow balloons for a party.
red balloons =  40 .
The number of green balloons Masha purchased = 3/5 of the red balloons.
The number of green balloons is again = 2/3 of yellow balloons.

What is the fraction?

Fraction is described as the number of compositions that comprise the Whole.

Here,
according to the question
Number of red ballons = 40
The number of green balloons Masha bought is 3/5 of the number of red balloons = 40 * 3/5
               = 8* 3
               = 24
The number of green balloons is also 2/3 of the number of yellow balloons.
= 3/2 * 24
= 36

Thus, the number of green and yellow balls is 24 and 36 respectively.

Learn more about fractions here:
https://brainly.com/question/10708469

#SPJ1

a question with a single correct answer
data that consists of numbers
a question that can have a variety of answers
:: statistical
III
:: numerical
nonstatistical

Answers

Answer:

a question with a single correct answer: non statistical

data that consists of numbers: numerical

a question that can have a variety of answers: statistical

Step-by-step explanation:

-

8x=10x+22
pls show work ty

Answers

X=11

Subtract 8x from both sides, this would be 2x+22 so x = 11.

Answer:

-11

Step-by-step explanation:

step 1

Move the variable to the left-hand side and change its sign

8x-10x=22

step 2

collect like terms

-2x=22

step 3

Divide both sides of the equation by -2

x=-11

Add -4 3/8 + 8 6/7 enter your answer

Answers

Answer:

Step-by-step explanation:

first you need to make the 3/8 and 6/7 into 21/56 and 48/56 then you take the -4 and 8 then add then together you get 4 then you still have the 21/56 and 48/56 which you need to add so 48 + 21 = 69/56 or 1 and 13/56 you then add the 1 to the 4 which gives you 5 then add the 13/56 to it which gives you your answer of 5 and 13/56


Describe a pair of independent events and a pair of dependent events. Explain your reasoning.

Answers

Independent events are the ones whose occurrence is unrelated to any other event.

Dependent events include those that are dependent on previous events.

What is described as independent events?

If the probability of an event A occurring is not impacted by the occurring of some other event B, then A & B are referred to as independent events.

Consider the example of a die roll. If A is the event 'the number appearing is odd' and B is the event 'the number showing up is a multiple of 3,'

P(A) = 3/6 = 1/2

P(B) = 2/6 = 1/3

A and B are also the events 'the number showing up is odd & a multiple of 3', implying that;

P(A ∩ B) = 1/6

P(A│B) = P(A ∩ B)/ P(B) = 1/2

Then P(A) = P(AB) = 1/2, implying that the happening of event B has had no effect on the probability of occurring of event A.

What is described as dependent events?

Dependent events include those that are dependent on previous events. The outcomes of previous events have an impact on these events. Dependent events are two or maybe more events that are dependent on one another.

When two events, A and B, are interdependent, the probability of their occurrence is:

P(A and B) = P(A) · P(B|A)

Assume we desire to have a queen.

The chances of obtaining a queen on the first draw are 4 from out 52 cards. If we receive a queen in the first draw, we have a 3 out of 51 chance of obtaining a queen in the second draw.

As a result, these are referred to as dependent events because the probability of a second event is determined by the results of the first draw.

To know more about the independent events, here

https://brainly.com/question/14106505

#SPJ4


In Δ J L P, m ∠J M P=3 x-6 J K=3 y-2 , and L K=5 y-8 .
Find L K if PK is a median.

Answers

In Δ JLP, m ∠ JMP = 3x - 6, JK = 3y - 2, and LK = 5y - 8 then the value of LK = 7.

What is a median?

The median is the value that separates the upper and lower halves of a data sample, population, or probability distribution. It is sometimes referred to as "the middle" value in a data set.

The middle number is discovered by sorting all data points and selecting the one in the middle (or if there are two middle numbers, taking the mean of those two numbers).

Since [tex]$\overline{P K}$[/tex] is a median, JK = KL.

3y - 2 = 5y - 8

simplifying the above equation, we get

3y - 5y - 2 = 5y - 5y - 8

-2y - 2 = -8

Adding 2 on both sides of the equation

-2y - 2 + 2 = -8 + 2

-2y = -6

y = 3

Substitute the value of y in LK, then we get

LK = 5y - 8

= 5(3) - 8

= 15 - 8

LK = 7

Therefore, the value of LK = 7.

To learn more about median refer to:

https://brainly.com/question/26151333

#SPJ4

Brooklyn has 3 shelves in her bedroom that have 40 books on each shelf in another room,she has 4 shelves that each contain 20 books if she buys 17 books at a resale shop, how many books does brooklyn have now

Answers

Answer:120 plus 80 plus 17 equals 217

Step-by-step explanation:

Answer:

Step-by-step explanation:

217

(c) 10y+12-7y-8-3y…..

Answers

Answer:

4

Step-by-step explanation:

Hello!

Simplify the expression by combining like terms.

Simplify10y + 12 - 7y - 8 - 3y10y - 7y - 3y + 12 - 8      Collect like terms3y - 3y + 4                      Combine like terms4                                     Simplify

The simplified expression is 4.

The simplified form of the given expression is 4.

To solve the expression 10y + 12 - 7y - 8 - 3y,

We need to combine like terms and simplify.

Simplify the expression by combining the like terms (the terms that have the same variable and the same exponent).

We have 10y, -7y, and -3y,

Which can be combined to give us a total of 0y.

This means that the variable y has been eliminated from the expression.

Combine the constant terms (the terms that don't have a variable or that have a variable raised to the power of 0).

We have 12 and -8, which can be combined to give us 4.

So, the simplified expression is 4.

We can express this mathematically as follows:

10y + 12 - 7y - 8 - 3y = (10y - 7y - 3y) + (12 - 8)

                                 = 0y + 4 = .

In other words, the expression simplifies to the constant term 4.

To learn more about the mathematical expression visit:

brainly.com/question/1859113

#SPJ6

If an analysis of variance is used for the following data, what would be the effect of changing the value of ss1 to 50? sample data m1 = 10 m2 = 20 ss1 = 90 ss2 = 70

Answers

The ​decreases SSwithin and increase the size of the F-ratio be the effect of changing the value of ss1 to 50 option (D) is correct.

What are the population and sample?

It is described as a collection of data with the same entity that is linked to a problem. The sample is a subset of the population, yet it is still a part of it.

It is given that:

Sample Data

M1 = 10 M2 = 15

SS1 = 90 SS2 = 70​

As we know,

The F ratio:

F = S²(x)/S²(y)

From the data given we can say:

Reduce SSwithin and boost the F-magnitude. ratio's

Thus, the ​decrease SSwithin and increase the size of the F-ratio be the effect of changing the value of ss1 to 50 option (D) is correct.

The question is incomplete.

The complete question is attached in the picture.

Learn more about the population and sample here:

brainly.com/question/9295991

#SPJ4

gina wilson unit 3 homework 1

Answers

My question and answer
Where is the question?
The answer is no idea

Solve for R
60 = -12r

Answers

The answer is the r = -5

simplify this with all the steps

Answers

Answer:

[tex]\frac{x}{x+2}[/tex]

Step-by-step explanation:

[tex]\frac{x^2+5x}{x^2+7x+10}[/tex] ← factorise numerator and denominator

= [tex]\frac{x(x+5)}{(x+5)(x+2)}[/tex] ← cancel (x + 5) on numerator / denominator

= [tex]\frac{x}{x+2}[/tex]

given x + y = -2 solve for y

Answers

Answer:

what is the value of x? Ans. varies

Step-by-step explanation:

Q is in the interior of

Answers

Answer:

...

Step-by-step explanation:

For every x ∈ R and each δ > 0 the interval (x−δ, x δ) carries both rational and irrational numbers. This implies that x can't be an interior factor of Q, and that x is a boundary point of Q. Therefore the indoors of Q is empty and the boundary of Q is R. Thus Q = b(Q) ∪ Q = R ∪ Q = R.

Q2 The length of a roll of ribbon is 30 metres, correct to the nearest half-metre. A piece of length 5.8 metres, correct to the nearest 10 centimetres, is cut from the roll. Work out the maximum possible length of ribbon left on the roll.​

Answers

The maximum possible length of ribbon left on the roll is 24.5 m.

Find bounds, Roll

30 → nearest half metre.

0.5 ÷ 2 = 0.25

LB = 30 – 0.25 = 29.75

GB = 30 + 0.25  = 30.25

Piece

5.8 →  nearest 10cm or 0.1m

(10÷100=0.1)

0.1÷2 = 0.05

LB = 5.8 – 0.05 = 5.75

GB = 5.8 + 0.05 = 5.85

Max length left = GB roll – LB piece

=30.25 – 5.75

=24.5m

The metric system's default unit of measurement for length. 100 centimeters equal to 1 metre. A meter is a common metric unit that is around 3 feet, 3 inches long. It measures 100 times more than a Centimetre, or 0.01 meters, in size. 1000 meters make up one kilometre.

Hence, the maximum possible length of ribbon left on the roll is 24.5 m.

To learn more about length of meters visit:

brainly.com/question/1606234

#SPJ1


Refer to ®R .
If SU = 16.2 centimeters, what is RT ?

Answers

The diameter of a circle is defined as being twice the length of its radius.

If SU = 16.2 centimeters then the value of RT = 8.1 cm.

What is the difference between the radius and diameter of a circle?

The diameter of a circle is defined as being twice the length of its radius. The radius of a circle is measured from the center to one of the circle's endpoints, whereas the diameter is measured from one end of the circle to another end of the circle, passing through the center.

In geometry, a circle's diameter is any straight line segment that passes through the center of the circle and has endpoints on the circle. It is also referred to as the circle's longest chord. Both definitions apply to a sphere's diameter.

The diameter is twice the radius because the greatest distance between two points on a circle must be a line segment through the center of the circle.

Here, SU is the diameter and RT is the radius.

Radius of the circle = 1/2 d

Where d = 16.2 then

substitute the value of d in the above equation, and we get

Radius of the circle  = 1/2 (16.2)

Radius of the circle = 8.1

Therefore, the value of RT = 8.1 cm.

The complete question is:

If SU = 16.2 centimeters, what is RT?

To learn more about the diameter refer to:

brainly.com/question/23220731

#SPJ4

PLEASE HELP
The Central Islip community has 9,649 homes in it. Smart Boards cost the school district $5,200 each. The HS needs 175, the Reed School needs 150 and the Mulligan school needs 50 new boards. Network Outsource the schools tech company has 12 workers for the HS, 8 for the Reed School and 4 for Mulligan. They all work 8 hours a day. They work 5 days a week, Monday thru Friday. They earn $58 per hour. It will take 45 weeks to finish the job. Find: a) Total Product Cost b) Total Labor Cost c) Total Cost d) Cost per Home e) Cost per Week

Answers

Using proportions, the costs are given as follows:

a) Total Labor Cost: $2,505,600.

b) Total Product Cost: $1,950,000.

c) Total Cost = $4,455,600.

d) Cost per home = $461.77.

e) Cost per week = $99,013.33.

What is a proportion?

A proportion is a fraction of a total amount, and the measures are related using a rule of three. Due to this, relations between variables, either direct(when both increase or both decrease) or inverse proportional(when one increases and the other decreases, or vice versa), can be built to find the desired measures in the problem, or equations to find these measures.

For item a, the labor cost is found using the earnings of the workers, as follows:

45 weeks x 5 days x 8 hours x 58 per hour x (12 + 8 + 4 workers)

Hence:

Total Labor Cost = 45 x 5 x 8 x 58 x 24 = $2,505,600.

For item b, the product cost is the cost of the boards, hence:

(175 + 150 + 50 boards) x 5,200

Total Product Cost = 375 x 5,200 = $1,950,000.

For item c, the total cost is the sum of the product cost and the labor cost, hence:

Total Cost = 2,505,600 + 1,950,000 = $4,455,600.

For item d, the cost per home is found dividing the total cost by the 9,649 homes, hence:

Cost per home = 4455600/9649 = $461.77.

For item e, the cost per week is found dividing the total cost by the 45 weeks, hence:

Cost per week = 4455600/45 = $99,013.33.

More can be learned about proportions at https://brainly.com/question/24372153

#SPJ1

c. Find the exact lengths of the altitudes of triangles B and C.
The length of the altitude of triangle B is
The length of the altitude of triangle C is

Answers

The right triangle B and c altitude length are same that is 6.

Given that,

In the given picture there are 2 right triangles.

They are B and C.

2 sides of the triangle B is 3 and [tex]3\sqrt{3}[/tex].

2 sides of the triangle C is 6 and [tex]6\sqrt{2}[/tex].

The angle for both the right triangle B and C is 90°.

We have to find the length of the altitude of right triangles B and C.

By using the Pythagorean theorem we can find the length of the altitude of the right triangle B and C.

The simplest way is to use the Pythagorean theorem if you are aware of two additional right triangle sides:

[tex]a^{2} +b^{2} =c^{2}[/tex]

Where the sides of the right triangle are a, b, and c.

First, We will find for the right triangle B.

Here, a=[tex]3\sqrt{3}[/tex] and b=3

Now, substitute in the formulae

[tex](3\sqrt{3})^{2} +3^{2}=c^{2}[/tex]

[tex]27+9=c^{2}[/tex]

[tex]36=c^{2} \\[/tex]

[tex]c^{2} =36\\[/tex]

Taking square root on both sides

[tex]\sqrt{c^{2} } =\sqrt{36}[/tex]

[tex]c=6[/tex]

Therefore, the length of the altitude of right triangle B is 6.

Now, We will find for the right triangle C.

Here, b=6 and c=[tex]6\sqrt{2}[/tex]

Now, substitute in the formulae

[tex]a^{2} +6^{2}=(6\sqrt{2}) ^{2}[/tex]

[tex]a^{2} +36=72[/tex]

[tex]a^{2}=72-36 \\[/tex]

[tex]a^{2} =36\\[/tex]

Taking square root on both sides

[tex]\sqrt{a^{2} } =\sqrt{36}[/tex]

[tex]a=6[/tex]

Therefore, the length of the altitude of right triangle B is 6.

To learn more about right triangle visit: https://brainly.com/question/6322314

#SPJ9

The formula for simple interest is I=Prt
a. Solve the formula for t, when r is the simple interest per year.
t=_
b. Use the new formula to find the value of t in the table.
The value of t in the table is _ years

Answers

a. The formula solved for t when r is the simple interest per year is t = I/Pr

b. Using the new formula, the value of t in the table is 3 years

Simple Interest

From the question, we are to solve the given formula for t

The given formula is

I = Prt

To solve for t, we will divide both sides of the equation by Pr

I/Pr = Prt/Pr

I/Pr = t

∴ t = I/Pr

b.

I = $75

P = $500

r = 5% = 0.05

t = 75/(500×0.05)

t = 75/25

t = 3 years

Hence, the value of t in the table is 3 years

Learn more on Simple interest here: https://brainly.com/question/20690803

#SPJ1


Which expression equals 5x/x² - 9 - 4 x / x² + 5x + 6?
(A) 7 x/(x-3)(x+3)(x+2) (B) x²-2 x / (x-3)(x+3)(x+2)
(C) x²+22x / x-3)(x+3)(x+2)
(D) 9x² - 2 x / (x-3)(x+3)(x+2)

Answers

The expression that is equal to [tex]\frac{5x}{x^{2} -9} -\frac{4x}{x^{2} +5x+6}[/tex] is [tex]\frac{x^{2}+22x }{(x+2)(x+3)(x-3)}[/tex].

In the given question

[tex]=\frac{5x}{x^{2} -9} -\frac{4x}{x^{2} +5x+6}[/tex].....(i)

First solving the denominator of both the terms ,

=x²-9

=x²-(3)²  

=(x+3)(x-3)    ...using identity a²-b²=(a+b)(a-b)   .....(ii)

=x²+5x+6

=x²+3x+2x+6       ...by splitting the middle term

=x(x+3)+2(x+3)

=(x+2)(x+3)       ....(iii)

Substituting the value of (ii) and (iii) in   (i)

we get,

[tex]=\frac{5x}{(x+3)(x-3)} -\frac{4x}{(x+2)(x+3) }[/tex]

taking LCM as (x+2)(x+3)(x-3) we get ,

[tex]=\frac{5x(x+2)-4x(x-3)}{(x+2)(x+3)(x-3)} \\ \\ =\frac{5x^{2} +10x-4x^{2} +12x}{(x+2)(x+3)(x-3)}[/tex]

Simplifying the like terms in the numerator we get

[tex]=\frac{x^{2} +22x}{(x+2)(x+3)(x-3)}[/tex]

Therefore , the expression that equals [tex]\frac{5x}{x^{2} -9} -\frac{4x}{x^{2} +5x+6}[/tex]  is [tex]\frac{x^{2} +22x}{(x+2)(x+3)(x-3)}[/tex].

The correct option is (c).

Learn more about Simplification here https://brainly.com/question/17198823

#SPJ4


What is an equation of each line?

Answers

The equation of line is ax + by + c = 0.

A line's equation has the conventional form ax + by + c = 0. Here, the variables are x and y, the coefficients are a and b, and the constant term is c. It is a first-order equation with the variables x and y. The coordinates of the point on the line shown in the coordinate plane are represented by the values of x and y.

With the aid of the line's slope and a point on the line, the equation of a line may be created. For a better understanding of how a line's equation is created, let's learn more about the line's slope and the necessary point on the line. The slope of a line is the angle that the line makes with the positive x-axis and can be stated as a numerical integer, fraction, or as the tangent of that angle. The term "point" refers to a coordinate system point with the x and y coordinates.

To learn more about equation of line from given link

https://brainly.com/question/18831322

#SPJ4

An investment portfolio is shown below.


Investment Amount Invested ROR
Savings Account $2,600 1.7%
Municipal Bond $3,700 3.2%
Preferred Stock $575 12.9%
Common Stock A $1,225 −5.6%


Using technology, calculate the weighted average ROR of the investments. Round to the nearest tenth of a percent.
2.1%
3.1%
5.9%
7.5%

Answers

Based on the investment portfolio given, the weighted average ROR of the investments would be 2.1%

What is the weighted average?

First, find the total portfolio value:

= 2,600 + 3,700 + 575 + 1,225

= $8,100

The weighted average Rate of Return (ROR) on the investments is:

= (2,600 / 8,100 x 1.7%) + (3,700 / 8,100 x 3.2%) + (575 / 8,100 x 12.9%) + (1,225 / 8,100 x -5.6%)

= 2.076

= 2.1%

Find out more on weighted average rate at https://brainly.com/question/28000301

#SPJ1

Answer:

2.1%

Step-by-step explanation:

I got it right.

Please answer///////////

Answers

Answer: the circle is not a function

the square is a function

Step-by-step explanation:

Other Questions
point x is located at (4.8, 0.8) and divides line segment AB in ratio 2:3 select two possible sets of endpoints for segment AB Is the firm maximizing profits? why or why not? identify a specific action that would increase this firm's profits. holding the cost of production constant, this firm should use Please help with the question on the photo :) 7. King Nebuchadnezzar rebuilt Nineveh as the center of theTrue or false? What is the price of 0.02percent of it is 0.01? on Tuesday the shop receives x cents by selling bottles of water at 45 cents each. In terms of x, how many bottles were sold If your car gets 33 miles per gallon, how much does it cost to drive 350 miles when gasoline costs $3.50 per gallon? Two people quit work and begin college at the same time. Their salary and education information is given in the table below.Salary prior to schoolYears attending collegeTotal cost of collegeSalary upon graduatingPerson A$18,0003$45000$33,000Person B$27,0004$30,000$37,000Choose the true statement.a.Person A recovers their investment in a shorter amount of time.b.Person B recovers their investment in a shorter amount of time.c.They recover their investments in the same amount of time.d.There is too little information to compare the time to recover their investments. Use the given information to determine whether LM is a perpendicular bisector, median, and/or an altitude of J K L . J L M K L M According to vsepr theory, which one of the following molecules is trigonal byramidal? a. sf4 b. xef4 c. nf3 d. sf6 e. pf5 Recent climate simulations suggest that angiosperms may have been necessary for the formation of tropical rainforests as we know them today. a. true b. false After a yellow solid is dissolved in water, the liquid appears to be uniformly yellow throughout. this yellow liquid is best classified as a(n):________ Why were some colonists discouraged by the state of religious practice in the colonies before the great awakening? How many contracts can you (safely) buy?b. if gold then declines $3.65 per troy ounce, what is the market-to-market equity in the account? What are text features? A Text features are the characters, setting, and plot.B Text features include headings, images, and glossaries. C Text features include predictions and different genres. D Text features are how the words are written.ANSWER QUICKLY PLS Tina's refrigerator unexpectedly failed and she had to purchase a replacement for $1,100. Unfortunately, this came at the same timethat she had to pay her real estate taxes. She decided to use the store's credit offer of a 1.2% monthly periodic rate if the balance is notpaid by the second month. If she pays $500 by the second month and plans to pay the balance due at the end of the fourth month, howmuch will she have to pay then?$619.87.$607.20.$614.49.$641.69. Help please??? Quick Suppose a balloon is filled with 5000 cm of helium. It then loses one fourth of its helium each day. How much helium will be left in the balloon at the start of the tenth day?c. How can you write a formula for this sequence? pleaseee help me someone Do you believe that the federal government has too much power or not enough? Please explain your answer.